Difference between revisions of "2024 AMC 12B Problems/Problem 13"

(Solution 2 (Coordinate Geometry and AM-QM Inequality))
(Solution 2 (Coordinate Geometry and AM-QM Inequality))
Line 29: Line 29:
 
~mitsuihisashi14
 
~mitsuihisashi14
  
==Solution 2 (Coordinate Geometry and AM-QM Inequality)==
+
==Solution 2 (Coordinate Geometry and AM-GM Inequality)==
  
 
[[Image:2024_amc_12B_P13_V2.PNG|thumb|center|500px|]]
 
[[Image:2024_amc_12B_P13_V2.PNG|thumb|center|500px|]]
Line 41: Line 41:
 
Note that they will be equal if and only if the circles are tangent,
 
Note that they will be equal if and only if the circles are tangent,
  
Applying the AM-QM inequality (<math> 2(a^2 + b^2) \geq (a+b)^2</math>) in the steps below, we get   
+
Applying the AM-GM inequality (<math> 2(a^2 + b^2) \geq (a+b)^2</math>) in the steps below, we get   
 
<cmath>
 
<cmath>
 
   h + k + 54 = (h + 25) + (k + 29) =\sqrt{(h + 25)}^2 + \sqrt{(k + 29)}^2 \geq \frac{\left(\sqrt{h + 25} + \sqrt{k + 29}\right)^2}{2}
 
   h + k + 54 = (h + 25) + (k + 29) =\sqrt{(h + 25)}^2 + \sqrt{(k + 29)}^2 \geq \frac{\left(\sqrt{h + 25} + \sqrt{k + 29}\right)^2}{2}
Line 50: Line 50:
  
  
~[https://artofproblemsolving.com/wiki/index.php/User:Cyantist luckuso]
+
~[https://artofproblemsolving.com/wiki/index.php/User:Cyantist luckuso],~[https://artofproblemsolving.com/wiki/index.php/User:ShortPeopleFartalot]
  
 
==Solution 3==
 
==Solution 3==

Revision as of 22:05, 25 December 2024

Problem 13

There are real numbers $x,y,h$ and $k$ that satisfy the system of equations\[x^2 + y^2 - 6x - 8y = h\]\[x^2 + y^2 - 10x + 4y = k\]What is the minimum possible value of $h+k$?

$\textbf{(A) }-54 \qquad \textbf{(B) }-46 \qquad \textbf{(C) }-34 \qquad \textbf{(D) }-16 \qquad \textbf{(E) }16 \qquad$


Solution 1 (Easy and Fast)

Adding up the first and second equation, we get: \begin{align*} h + k &= 2x^2 + 2y^2 - 16x - 4y \\ &= 2(x^2 - 8x) + 2(y^2 - 2y) \\  &= 2(x^2 - 8x) + 2(y^2 - 2y) \\ &= 2(x^2 - 8x + 16) - (2)(16) + 2(y^2 - 2y + 1) - (2)(1) \\ &= 2(x - 4)^2 + 2(y - 1)^2 - 34 \end{align*} All squared values must be greater than or equal to $0$. As we are aiming for the minimum value, we set the two squared terms to be $0$.

This leads to $\min(h + k) = 0 + 0 - 34 = \boxed{\textbf{(C)} -34}$

~mitsuihisashi14

Solution 2 (Coordinate Geometry and AM-GM Inequality)

2024 amc 12B P13 V2.PNG

\[(x-3)^2 + (y-4)^2 = h + 25\] \[(x-5)^2 + (y+2)^2 = k + 29\] The distance between 2 circle centers is \[(O_{1}O_{2})^2 = (5-3)^2 + (4 - (-2)) ^2 = 40\] The 2 circles must intersect given there exists one or more pairs of (x,y), connecting $O_{1}O_{2}$ and any pair of the 2 circle intersection points gives us a triangle with 3 sides, then \[radius (O_{1}) + radius (O_{2}) \geq O_{1}O_{2}\] \[\sqrt{h+25} + \sqrt{k+29}   \geq  2\cdot\sqrt{10}\] Note that they will be equal if and only if the circles are tangent,

Applying the AM-GM inequality ($2(a^2 + b^2) \geq (a+b)^2$) in the steps below, we get \[h + k + 54 = (h + 25) + (k + 29) =\sqrt{(h + 25)}^2 + \sqrt{(k + 29)}^2 \geq \frac{\left(\sqrt{h + 25} + \sqrt{k + 29}\right)^2}{2} \geq   \frac{\left(2\sqrt{10}\right)^2}{2} = 20.\]

Therefore, $h + k \geq 20 - 54 = \boxed{C -34}$.


~luckuso,~[1]

Solution 3

2024 AMC 12B P13.jpeg

~Kathan

Video Solution 1 by SpreadTheMathLove

https://www.youtube.com/watch?v=U0PqhU73yU0

See also

2024 AMC 12B (ProblemsAnswer KeyResources)
Preceded by
Problem 12
Followed by
Problem 14
1 2 3 4 5 6 7 8 9 10 11 12 13 14 15 16 17 18 19 20 21 22 23 24 25
All AMC 12 Problems and Solutions

The problems on this page are copyrighted by the Mathematical Association of America's American Mathematics Competitions. AMC logo.png